How many 5 card hands are possible with four black cards and one red card?

Answers

Answer 1

Answer:

1 i believe

Step-by-step explanation:


Related Questions

What is the square root of 91

Answers

Answer:

9.53939201417

Step-by-step explanation:

sqrt(91)=

9.53939201417

Answer:

9.53939201

Step-by-step explanation:

This can simply be done in a calculator but what you are basically doing is thinking about the reverse of a exponent. For instance, √25 is 5 because 5² is 25. Hope this helps! If you are still confused please comment!

Given the function f(x)=7x-4 find f(3)

Answers

Answer:

17

Step-by-step explanation:

f(x)=7(3)-4     Distributive Property

f(x)=21-4        Subtract

f(x)=17

The value of f(3) is 17

what is function?

Function, an expression, rule, or law that defines a relationship between one variable (the independent variable) and another variable (the dependent variable). Functions are ubiquitous in mathematics and are essential for formulating physical relationships in the sciences.

Given:

f(x)=7x-4

Now for x=3

f(3)= 7*3-4

=21-4

=17

Hence, the value of function is 17.

Learn more about function here:

https://brainly.com/question/12431044

#SPJ2

Quadrilateral ABCD is inscribed in a circle
What is the measure of angle A?

Answers

Answer:

Quadrilateral ABCD is inscribed in a circle, then

A + C = 180

=> 2x + 9 + 3x + 1 = 180

=> 5x = 170

=> x = 34

=> A  = 2 x 34 + 9 = 77 deg

Hope this helps!

:)

Answer:

77°

Step-by-step explanation:

ABCD is inscribed in a circle. Therefore it is a cyclic quadrilateral. Opposite angles of a cyclic quadrilateral are supplementary.

[tex]\therefore \angle A + \angle C = 180° \\ (2x + 9) \degree + (3x + 1) \degree =180° \\ (5x + 10) \degree =180° \\ 5x + 10 = 180 \\ 5x = 180 - 10 \\ 5x = 170 \\ x = \frac{170}{5} \\ x = 34 \\ \angle A = (2 \times 34 + 9) \degree \\ \huge \red{ \boxed{\angle A= 77 \degree}} \\ [/tex]

Which relation is a function? A. (1, 0), (3, 0), (1, 1), (3, 1) (1, 3) B. (1, 1), (2, 2), (3, 3), (4, 4), (5, 8) C. (2, 7), (6, 5), (4, 4), (3, 3), (2, 1) D. (9, −3), (9, 3), (4, −2), (4, 2), (0, 0)

Answers

Answer:

b is the answer

Step-by-step explanation:

i checked if inputs repeated in all the string of answers

Answer:

b

Step-by-step explanation:

Which expression represents the volume of a rectangular prism that is not occupied by the cylinder.

Answers

Answer:

The volume of the prism not occupied by the cylinder is (150 - 25*pi)*h.

Step-by-step explanation:

In order to calculate the volume of the rectangular prism that is not occupied by the cylinder we need to calculate the total volume of the rectangular prism and subtract it by the volume of the cylinder. We have:

Rectangular prism:

Volume prism = length*height*width

Volume prism = 15*h*10 = 150*h

Cylinder:

Volume cylinder = pi*r²*h = pi*5²*h = 25*pi*h

The volume of the prism that is not occupied by the cylinder is:

volume not occupied = volume prism - volume cylinder

volume not occupied = 150*h - 25*pi*h = (150 - 25*pi)*h

The volume of the prism not occupied by the cylinder is (150 - 25*pi)*h.

factor the polynomial 8x-2

Answers

Answer:

[tex]2(4x-1)[/tex]

Step-by-step explanation:

you divide by 2 and simplify. idk how else to explain it....

Answer:

2(4x-1)

Step-by-step explanation:

Factor out a 2

Evaluate 9 − b 9−b9, minus, b when b = 8b=8b, equals, 8.

Answers

Answer:

the way you wrote the question is confusing. Please right it in better wording or with line breaks

Step-by-step explanation:

Suppose you start an annuity where you invest $2,000 at the beginning of each year and 4% interest is paid at the end of the year. What is the value of the annuity at the end of 5 years, rounded to the nearest dollar? It is $

Answers

I believe $80 dollars would be 4% of 2,000

The value of the annuity at the end of 5 years, rounded to the nearest dollar is $10,833.

What is the value of the annuity at the end of 5 years?

The formula that can be used to determine the future value of the annual annuity is:

Yearly deposity x annuity factor

Annuity factor = {[(1+r)^n] - 1} / r

Where:

r = interest rate n = number of years

2000 x {[(1.04^5) - 1] / 0.04} = $10,832.65

To learn more about annuites, please check: https://brainly.com/question/24108530

Given a school supply store sells pencils for $2, pens for $4, and markers for $5. Write an expression for the total cost of 3 pencils, 2 pens, and 3 markers

Answers

Answer: 29

Step-by-step explanation:

3x2= 6

4x2=8

3x5=15

6+8+15= 29

hoped I helped you a bit! Good luck! :)

A pink crayon is made with 12\text{ mL}12 mL12, start text, space, m, L, end text of red wax for every 5\text{ mL}5 mL5, start text, space, m, L, end text of white wax

Answers

Answer:

Your answer would be:

#1: 36 mL of red wax mixed with 15 mL of white wax.

#2: 60 mL of red wax mixed with 25 mL of white wax.

HOPE THIS HELP

Answer:c and b

Step-by-step explanation:

What is the volume of a rectangular prism with a length of 2 inches, a width of an inch & is a quarter of an inch in height?

Answers

Answer:

= 1/2 inches ^3

Step-by-step explanation:

The volume of a prism

V = l*w*h

  = 2 * 1 *1/4

  = 1/2 inches ^3

Answer:

[tex] = \frac{1}{2} {in}^{3} [/tex]

Step-by-step explanation:

[tex]v = whl \\ = 1 \times \frac{1}{4} \times 2 \\ = \frac{2}{4} \\ = \frac{1}{2} {in}^{3} [/tex]

hope this helps

brainliest appreciated

good luck! have a nice day!

James has g amount of games. Samuel, his friend, has 4 times as many games as James. Write an algebraic expression to represent how many games James has and how many games Samuel has in terms of g.

Answers

Answer: The answer is given below

Step-by-step explanation:

From the question, we are told that James has a lot of games and his friend Samuel, has 4 times as many games as James.

So let the number of games that James has be denoted by g.

Since Samuel has 4 times as the number of games that James has, then Samuel's game will be

= 4 × g

= 4g

Therefore, the expression for James will be g and the expression for the number of games Samuel has is 4g.

Describe two methods that can be used to find the area of the composite figure.

Answers

Answer: A composite figure is made up of several simple geometric figures such as triangles, rectangles, squares, circles, and semicircles. To find the area of a composite figure, separate the figure into simpler shapes whose area can be found. Then add the areas together.

Area of the composite figure can be find by using additive areas method and subtractive areas method

What is Composite figure?

Composite geometric figures are made from two or more geometric figures. The composite figure below is made from parts of a rectangle and a circle

What is Area ?

Area is the quantity that expresses the extent of a region on the plane or on a curved surface.

Area of the composite figure can be find by using two methods,

Additive areas method and Subtractive areas method

In Addictive areas method

Break the composite shape into basic shapesFind the area of the each basic shapesAdd all the areas of basic shapes together

In Subtractive areas method

Break the composite shape into basic shapes. That shape maybe larger than the composite shapes.Find the area of the each basic shapesArea of the composite shape will be the difference between the area of the larger shape.

Hence, the area of  the composite figure can be find by using additive areas method and subtractive areas method

Learn more about Composite figure and Area here

https://brainly.com/question/16193772

#SPJ2

A triangle has a side with length of 6 feet and another side with length of 8 feet. The angle between the sides measures 73°. Find the area of the triangle. Round your answers to the nearest tenth. Please show work.
a. 23.0 ft^2
b. 45.9 ft^2
c. 1752 ft^2
d. 7.0 ft^2

Answers

Answer:

Area = (1/2) * side 1 * side 2 * sine (73 degrees)

Area = (1/2) * 6 *  8 * .9563

Area = 22.9512  square feet

Step-by-step explanation:

Answer:

a

Step-by-step explanation:

Area = ½ a b sinC

= ½ × 6 × 8 × sin(73)

= 22.9513141431

To the nearest tenth: 23.0 ft²

I will mark brainliest if you are correct!
When solving the equation, which is the best first step to begin to simplify the equation?
-2 (x + 3) = -10

A. (-2) (-2) (x + 3) = -10 (-2)
B. -1/2 (-2) (x + 3) = -10 (-1/2)
C. -2/2 (x + 3) = -10/2
-2/-10 (x + 3) = -10/-10

Answers

Answer:

i think the answer is C

Step-by-step explanation:

Answer:

Look below.

Step-by-step explanation:

Uh here's what I did:

-2x-6= -10

Add 6 to both sides.

-2x=-4

Divide both sides by -2

x=2

The length of a rectangular plot is expressed as x²m and the breadth is xm if the area of the rectangular plot is 5832sqcm then find it's length and breadth

Answers

Answer:

Length = x^2 = 0.835^2 = 0.698 m = 69.8cm

Breadth = x = 0.835 m = 83.5cm

Step-by-step explanation:

Given;

Length of the rectangular plot l = x^2 m

Breadth b = x m

Area A = 5832 cm^2 = 0.5832 m^2

(They should be converted to the same units before solving: length in metres and area in square metres)

The area of a rectangular plot can be calculated using the formula;

Area = length × breadth

A = l × b

Substituting the values of l and b;

A = x^2 × x

A = x^3

Substituting the value of A;

0.5832 = x^3

solving for x;

x = (0.5832)^(1/3)

x = 0.835 m

Length = x^2 = 0.835^2 = 0.698 m = 69.8cm

Breadth = x = 0.835 m = 83.5cm

There are 110 people at a meeting. They each shake hands with everyone else. How many handshakes were there?

A) Permutation

B) Combination

C) Circular permutation

Answers

Each of them shakes hands with 109 others.
109*110 = 11990 handshakes were done.

Please help me now plz I will mark you brainliest

Answers

Answer:

Neither

Step-by-step explanation:

It can be paralel because they both pass through -7 and it cant be perpendicular because the dont intersect at a 90 degree angle

PLEASE HELP ASAP 10 POINTS AND ITS MULTIPLE CHOICE

Which equation represents the line that is parallel to the following?
Y=-3x-4

A. Y=3x+4
B. Y=3x+1
C. Y=-1/3x-4/3
D. Y=1/3x-4

Answers

Answer:

A. Y=3x+4

Step-by-step explanation:

Pleaaase help me out here im being timed!!

Really appreciate it if u did thank u.

Answers

The answer is 180. Or C.A triangle’s full angle is 180 so the answer is 180

8+16÷4·3+5·4·25·2 =

solve the equation

Answers

Answer:

1020

Step-by-step explanation:

8+16÷4·3+5·4·25·2

PEMDAS

Multiply and divide from left to right

8+4·3+5·4·25·2

8+12+5·4·25·2

8+12+1000

Then add

1020

1. Si tengo el siguiente ejercicio x = 8+9*4+(32*2+5*4)+3*6 aplicando la jerarquía de operaciones, el resultado sería? a. 25 b. 45 c. 78 d. Ninguna de las anteriores

Answers

Answer:

Option D is correct.

The answer is none of the above.

x = 146

La opción D es correcta.

La respuesta es ninguna de las anteriores.

x = 146

Step-by-step explanation:

English Translation

If I have the following exercise x = 8 + 9 * 4 + (32 * 2 + 5 * 4) + 3 * 6 applying the hierarchy of operations, the result would be?

a. 25

b. 45

c. 78

d. None of the above

Solution

The hierarchy of operations includes BODMAS.

B - Brackets

O - Order

D - Division

M - Multiplication

A - Addition

S - Subtraction

x = 8 + 9 * 4 + (32 * 2 + 5 * 4) + 3 * 6

Solving the bracket first.

The multiplication in the bracket are first simplified to give

x = 8 + 9 * 4 + (64 + 20) + 3 * 6

Then further simplifying the bracket

x = 8 + 9 * 4 + (84) + 3 * 6

We then solve the multiplication outside the brackets

x = 8 + 36 + 84 + 18

We now sum all of this

x = 146

In Spanish/En Español

La jerarquía de operaciones incluye BODMAS.

B - Soportes

O - Orden

D - División

M - Multiplicación

A - Suma

S - Resta

x = 8 + 9 * 4 + (32 * 2 + 5 * 4) + 3 * 6

Resolver el soporte primero.

La multiplicación en el paréntesis se simplifica primero para dar

x = 8 + 9 * 4 + (64 + 20) + 3 * 6

Luego simplificando aún más el soporte

x = 8 + 9 * 4 + (84) + 3 * 6

Luego resolvemos la multiplicación fuera de los corchetes

x = 8 + 36 + 84 + 18

Ahora sumamos todo esto

x = 146

Hope this Helps!!!

¡¡¡Espero que esto ayude!!!

In ΔHIJ, the measure of ∠J=90°, IJ = 47 feet, and HI = 68 feet. Find the measure of ∠H to the nearest degree.

Answers

Answer:

44

Step-by-step explanation:

Describe in your own words what a hyperbola is.

Answers

Answer:

A conic section that can be thought of as an inside-out ellipse

Step-by-step explanation:

Hyperbola is a two-branched open curve-conic section.

What is a hyperbola?

Hyperbola is the locus of all those points in a plane such that the difference in their distances from two fixed points in the plane is a constant.

For the given situation,

We need to describe about a hyperbola.

A hyperbola is two curves that are like infinite bows. It is a two-branched open curve, a  conic section produced by the intersection of a circular cone and a plane that cuts both nappes of the cone.

The standard equation of hyperbola is

                     [tex][\frac{x^{2} }{a^{2} }-\frac{y^{2} }{b^{2} }]=1[/tex]

where,[tex]b^2 = a^2 (e^2 -1)[/tex]

a,b are the points on the plane and e is the eccentricity.

Hence we can conclude that hyperbola is a two-branched open curve-conic section.

Learn more about hyperbola here

https://brainly.com/question/17090445

#SPJ2

Write nine minus the quotient of two and a number x as a expression variable

Answers

Answer:

9-(2/x)

Step-by-step explanation:

helppppppppppppp please

Answers

Answer:

with?

Step-by-step explanation:

Simplify 8^2/3 -√9×10^0+(1/144)^-1/2

Answers

Answer:

13

Step-by-step explanation:

We have the following expression:

[tex]8^{\frac{2}{3} } -\sqrt{9}*10^{0} + (\frac{1}{144}) ^{-\frac{1}{2} }[/tex]

If we do part by part, we have remains:

[tex]8^{\frac{2}{3} }[/tex] = 4

8 = 2^3

Therefore, the exponents 3 * 2/3 = 2 are multiplied

2^2 = 4

[tex]\sqrt{9}*10^{0}[/tex] = 3

We know that the root of 9 is equal to 3 and that every number raised to 0 is equal to 1, therefore 1 * 3 = 3

[tex](\frac{1}{144}) ^{-\frac{1}{2} }[/tex] = 12

When the exponent is negative, the fraction can be inverted and at the end there is 144 ^ 1/2, which would be the root of 144, which is equal to 12.

So, we have:

4 - 3 + 12 = 13

Solve the following system of equations {4x−6y=105x+6y=−55

Answers

Answer:

31557778998765432

Step-by-step explanation:

is it impossible

A bag contains 4 red, 2 blue, 6 green, and 8 white marbels. Round anwsers to the nearest tenth. What is the probibility of selecting a green marble, replacing it, and then selecting a red marble.

Answers

Answer:

[tex]\frac{3}{50}[/tex] chance.

Step-by-step explanation:

Probability of getting a green: [tex]\frac{6}{4+2+6+8}[/tex] = [tex]\frac{6}{20} = \frac{3}{10}[/tex]

Probability of getting red: [tex]\frac{4}{4+2+6+8} = \frac{4}{20} = \frac{1}{5}[/tex]

[tex]\frac{1}{5} * \frac{3}{10} = \frac{3}{50}[/tex]

There is a [tex]\frac{3}{50}[/tex] chance of getting a green then red marble with replacement.

please help immediately very confused

Answers

Answer:

4.5

Step-by-step explanation:

162/360=0.45

0.45*10=4.5

Answer:

4.5units

Step-by-step explanation:

arc length's formula:2πr(∆/360)

remember, 2πr: circumference of a circle:

10units according to diagram.

∆=162°

using formula, 10(162°/360)

10*162/360 : 162/36

:4.5 units.

Other Questions
In which of the following situations would a sound editor use a cross fade?a wedding sequence is being filmed from multiple anglesoa lead character is walking out of a festive party and following a mysterious character into the nighta montage sequence is being cut for a new product commercialO a car chase scene is being filmed with both dolly and mounted camera systems How are the solution sets to s HELP ME!!!!!!!!!!!!!!!Select the correct answer.How would you describe the child in Elizabeth Bishop's "Sestina"?A. unrulyB. spoiledC. imaginativeD. disrespectful Javier is 175% heavier than his brother. If Javiers brother weighs 80 pounds, how much does Javier weigh? The following information was drawn from the balance sheets of the Kansas and Montana companies: Kansas Montana Current assets $ 59,000 $ 78,000 Current liabilities 40,000 43,000 Required a. Compute the current ratio for each company. b. Which company has the greater likelihood of being able to pay its bills? c. Assume that both companies have the same amount of total assets. Speculate as to which company would produce the higher return-on-assets ratio. Connection to modern world from the myth narcissus and echo 1. Which of the following most directly caused the Cold War? (1 point)fighting for control of outer spaceformation of the United Nationsdifferent economic and political ideasreligious differences among nations2. Which of the following did many people fear during the Cold War? (1 point)droughtstarvationnuclear wareconomic depression3. Which of the following most contributed to the increase of Southeast Asian immigration to theThe United States in the late 1900s?(1 pointregional conflictsfamine and diseasetougher immigration Please help me with this question Pls answer the last question thank you! Number 5. Which statements explain how Roosevelt use of illusions most likely affect her audience Provide the reflected ordered pair for each of the given ordered pairs. Reflect the ordered pair ( 4 , 5 ) across the y-axis Reflect the ordered pair ( 0 , 3 ) across the x-axis The average "capitalist" often believes the principle of "laissez-faire", which means let it be. Which economy has this idea at its core? a) Command Economyb) Traditional Economyc) Mixed Economyd) Free Market Economy Elliott has some yarn that she wants to use to make hats and scarves. Each hat uses 0.2, point, 2 kilograms of yarn and each scarf uses 0.1, point, 1 kilograms of yarn. Elliott wants to use twice as much yarn for scarves as for hats, and she wants to make a total of 20 items. Let h be the number of hats Elliott makes and s be the number of scarves she makes. Which system of equations represents this situation? 49 increased by twice a number m In the diagram below, OPis circumscribed about quadrilateral ABCD. What isthe value of x?A. 80oo120-B. 30C. 120O OD. 60SUEM MES< PREVIOUS An mp3 player is on sale for 30% off the regular price of $39.98. How much is the discount? pls ASAP! giving BRAINLIEST if you are right! what roles does the court system have in our society 4688-3) -30X-5 CX-5) Ameeta finished 12 math problems. David finished 3 times as many. How many more math problems did David finish than Ameeta?